MGMAT Sequence

This topic has expert replies
User avatar
Master | Next Rank: 500 Posts
Posts: 112
Joined: Thu Mar 22, 2012 9:33 am
Thanked: 1 times
Followed by:1 members

MGMAT Sequence

by rommysingh » Thu Sep 17, 2015 2:16 pm
The next number in a certain sequence is defined by multiplying the previous term by some positive constant k, where k ≠ 1. How many of the first nine terms in this sequence are greater than 1?

(1) The ninth term in this sequence is 81.

(2) The fifth term in this sequence is 1.

GMAT/MBA Expert

User avatar
Elite Legendary Member
Posts: 10392
Joined: Sun Jun 23, 2013 6:38 pm
Location: Palo Alto, CA
Thanked: 2867 times
Followed by:511 members
GMAT Score:800

by [email protected] » Thu Sep 17, 2015 10:16 pm
Hi rommysingh,

This DS question tells us that each term in a sequence is equal to the PREVIOUS term multiplied by a POSITIVE CONSTANT.

For example, the sequence 1, 2, 4, 8, 16 would fit this definition (another example would be 16, 8, 4, 2, 1, 1/2, etc.). We don't know any of the terms though and we don't know the constant (it could be either an integer, fraction or mixed number). We DO know that since we're multiplying by a positive constant that the sequence of numbers either "increases" or "decreases."

We're asked how many of the first 9 terms are greater than 1? It's interesting that the question asks how many are greater than 1.

Fact 1: the 9th term = 81

If the constant is 3, then the terms are....
1/81, 1/27, 1/9, 1/3, 1, 3, 9, 27, 81
and 4 terms are greater than 1.

If the constant is 1/2, then the terms are....
(Four big numbers that decrease by 50% each time), 1296, 648, 324, 162, 81
and ALL 9 terms are greater than 1.

Fact 1 is INSUFFICIENT

Fact 2: the 5th term = 1

Using the same logic that we used in Fact 1, since the sequence either increases or decreases, we could have...

A series that increases UP to 1, then increases past it....

Four numbers less than 1, 1, Four numbers greater than 1

Or

A series the decreases DOWN to 1, then decreases past it...

Four numbers greater than 1, 1, Four numbers less than 1

Regardless of which option, we end up with exactly 4 terms that are GREATER than 1.
Fact 2 is SUFFICIENT

Final Answer: B

GMAT assassins aren't born, they're made,
Rich
Last edited by [email protected] on Fri Sep 18, 2015 10:26 am, edited 3 times in total.
Contact Rich at [email protected]
Image

User avatar
Master | Next Rank: 500 Posts
Posts: 112
Joined: Thu Mar 22, 2012 9:33 am
Thanked: 1 times
Followed by:1 members

by rommysingh » Fri Sep 18, 2015 10:14 am
Hi Rich,
thanks for reverting. It says fact 1 - 9th term is 81 so

term shd be some constant on term 9 is 81 where as in your example you state the second term to be 81 itself 81. I do get the logic of number increase or decrease but its difficult when you show in fact 1 and two. can you please show in examples ....

Fact 1: the 9th term = 81

If the terms are 9, 81, 729, etc., then ALL 9 terms are greater than 1.
If the terms are 1, 81, 81(81), etc. then 8 of the germs are greater than 1.
Fact 1 is INSUFFICIENT

Fact 2: the 5th term = 1

Since the sequence either increases or decreases, we'd have...
4 numbers less than 1, 1, 4 numbers greater than 1
or
4 numbers greater than 1, 1, 4 numbers less than 1

Regardless of which option, we end up with exactly 4 terms.
Fact 2 is SUFFICIENT

GMAT/MBA Expert

User avatar
Elite Legendary Member
Posts: 10392
Joined: Sun Jun 23, 2013 6:38 pm
Location: Palo Alto, CA
Thanked: 2867 times
Followed by:511 members
GMAT Score:800

by [email protected] » Fri Sep 18, 2015 10:33 am
Hi rommysingh,

I've added more specific examples into my original post.

GMAT assassins aren't born, they're made,
Rich
Contact Rich at [email protected]
Image

User avatar
Elite Legendary Member
Posts: 3991
Joined: Fri Jul 24, 2015 2:28 am
Location: Las Vegas, USA
Thanked: 19 times
Followed by:37 members

by Max@Math Revolution » Sat Sep 19, 2015 11:38 pm
Forget conventional ways of solving math questions. In DS, Variable approach is the easiest and quickest way to find the answer without actually solving the problem.
Remember equal number of variables and independent equations ensures a solution.

The next number in a certain sequence is defined by multiplying the previous term by some positive constant k, where k ≠ 1. How many of the first nine terms in this sequence are greater than 1?

(1) The ninth term in this sequence is 81.

(2) The fifth term in this sequence is 1.

There are 2 variables in the original condition (the initial value and variable k). Therefore in order to match the number of equations and variables, we need 2 equations. Since there is 1 each in 1) and 2) C has high probability of being the answer. Using both 1) & 2) together, 9-th=k^4=81 gives us k=3 and therefore 5-th=3, 6-th=9, 7-th=27, 8-th=81. Since 4 is larger than 1, the conditions are sufficient and therefore the answer is C. However in tip 4, we can't have a trivial case as our answer and therefore we should try using 1) and 2) separately.

In case of 1), we don't know the value of k. therefore the condition is not sufficient.
In case of 2), when k>1, 1/k^4, 1/k^3, 1/k^2, 1/k,1,k,k^2,k^3,k^4 and therefore the 4 latter terms are greater than 1. When 0<k<1, the former 4 terms are greater than 1, therefore the answer is unique. Thus the answer is B.


Normally for cases where we need 2 more equations, such as original conditions with 2 variable, or 3 variables and 1 equation, or 4 variables and 2 equations, we have 1 equation each in both 1) and 2). Therefore C has a high chance of being the answer, which is why we attempt to solve the question using 1) and 2) together. Here, there is 70% chance that C is the answer, while E has 25% chance. These two are the key questions. In case of common mistake type 3,4, the answer may be from A, B or D but there is only 5% chance. Since C is most likely to be the answer according to DS definition, we solve the question assuming C would be our answer hence using ) and 2) together. (It saves us time). Obviously there may be cases where the answer is A, B, D or E.

Math Revolution : Finish GMAT Quant Section with 10 minutes to spare
The one-and-only World's First Variable Approach for DS and IVY Approach for PS with ease, speed and accuracy.
Unlimited Access to over 120 free video lessons - try it yourself (https://www.mathrevolution.com/gmat/lesson)
See our Youtube demo (https://www.youtube.com/watch?v=R_Fki3_2vO8)